Difference between revisions of "2014 AMC 12B Problems/Problem 20"

(Solution)
Line 7: Line 7:
 
\textbf{(D) }20\qquad
 
\textbf{(D) }20\qquad
 
\textbf{(E) }\text{infinitely many}\qquad</math>
 
\textbf{(E) }\text{infinitely many}\qquad</math>
 
== Video Solution ==
 
https://youtu.be/RdIIEhsbZKw?t=1088
 
 
~ pi_is_3.14
 
  
 
==Solution==
 
==Solution==
Line 20: Line 15:
 
<cmath>x \not = 50</cmath>
 
<cmath>x \not = 50</cmath>
 
Hence, we have integers from 41 to 49 and 51 to 59. There are <math>\boxed{\textbf{(B)} 18}</math> integers.
 
Hence, we have integers from 41 to 49 and 51 to 59. There are <math>\boxed{\textbf{(B)} 18}</math> integers.
 +
 +
== Video Solution ==
 +
https://youtu.be/RdIIEhsbZKw?t=1088
 +
 +
~ pi_is_3.14
  
 
== See also ==
 
== See also ==
 
{{AMC12 box|year=2014|ab=B|num-b=19|num-a=21}}
 
{{AMC12 box|year=2014|ab=B|num-b=19|num-a=21}}
 
{{MAA Notice}}
 
{{MAA Notice}}

Revision as of 08:16, 27 January 2021

Problem

For how many positive integers $x$ is $\log_{10}(x-40) + \log_{10}(60-x) < 2$ ?

$\textbf{(A) }10\qquad \textbf{(B) }18\qquad \textbf{(C) }19\qquad \textbf{(D) }20\qquad \textbf{(E) }\text{infinitely many}\qquad$

Solution

The domain of the LHS implies that \[40<x<60\] Begin from the left hand side \[\log_{10}[(x-40)(60-x)]<2\] \[-x^2+100x-2500<0\] \[(x-50)^2>0\] \[x \not = 50\] Hence, we have integers from 41 to 49 and 51 to 59. There are $\boxed{\textbf{(B)} 18}$ integers.

Video Solution

https://youtu.be/RdIIEhsbZKw?t=1088

~ pi_is_3.14

See also

2014 AMC 12B (ProblemsAnswer KeyResources)
Preceded by
Problem 19
Followed by
Problem 21
1 2 3 4 5 6 7 8 9 10 11 12 13 14 15 16 17 18 19 20 21 22 23 24 25
All AMC 12 Problems and Solutions

The problems on this page are copyrighted by the Mathematical Association of America's American Mathematics Competitions. AMC logo.png